Difference between revisions of "2002 AMC 12A Problems/Problem 15"

(Note)
(Video Solution)
(13 intermediate revisions by 8 users not shown)
Line 17: Line 17:
 
</math>
 
</math>
  
== Solution ==
+
== Solution 1 ==
  
 
As the unique mode is <math>8</math>, there are at least two <math>8</math>s.
 
As the unique mode is <math>8</math>, there are at least two <math>8</math>s.
Line 25: Line 25:
 
If the largest one is <math>16</math>, then the smallest one is <math>8</math>, and thus the mean is strictly larger than <math>8</math>, which is a contradiction.
 
If the largest one is <math>16</math>, then the smallest one is <math>8</math>, and thus the mean is strictly larger than <math>8</math>, which is a contradiction.
  
If the largest one is <math>15</math>, then the smallest one is <math>7</math>. This means that we already know four of the values: <math>8</math>, <math>8</math>, <math>7</math>, <math>15</math>. Since the mean of all the numbers is <math>8</math>, their sum must be <math>64</math>. Thus the sum of the missing four numbers is <math>64-8-8-7-15=26</math>. But if <math>7</math> is the smallest number, then the sum of the missing numbers must be at least <math>4\cdot 7=28</math>, which is again a contradiction.
+
If we have 2 8's we can add find the numbers 4, 6, 7, 8, 8, 9, 10, 12.
 +
This is a possible solution but has not reached the maximum.
  
If the largest number is <math>14</math>, we can easily find the solution <math>(6,6,6,8,8,8,8,14)</math>. Hence, our answer is <math>\boxed{\text{(D)}\ 14 }</math>.
+
If we have 4 8's we can find the numbers 6, 6, 6, 8, 8, 8, 8, 14.
  
===Note===
+
We can also see that they satisfy the need for the mode, median, and range to be 8. This means that the answer will be
 +
<math>\boxed{\text{(D)}\ 14 }</math>. ~By QWERTYUIOPASDFGHJKLZXCVBNM
  
The solution for <math>14</math> is, in fact, unique. As the median must be <math>8</math>, this means that both the <math>4^\text{th}</math> and the <math>5^\text{th}</math> number, when ordered by size, must be <math>8</math>s. This gives the partial solution <math>(6,a,b,8,8,c,d,14)</math>. For the mean to be <math>8</math> each missing variable must be replaced by the smallest allowed value.
+
== Solution 2 ==
The solution that works is <math>(6,6,6,8,8,8,8,14)</math>.
+
 
 +
We could express this collection as integers <math>a\textsubscript{1}</math> through <math>a\textsubscript{8}</math>, with <math>a\textsubscript{1}</math> being the smallest and <math>a\textsubscript{8}</math> being the largest.
 +
 
 +
Since the mean is <math>8</math>, we know that <math>a\textsubscript{4}</math> and <math>a\textsubscript{5}</math> must also be <math>8</math>. If they were not, the other numbers, which are lesser and greater than <math>a\textsubscript{4}</math> and <math>a\textsubscript{5}</math> respectively, would not be able to satisfy the condition that <math>8</math> is the mode.
 +
 
 +
There are <math>8</math> terms and the mean is <math>8</math>. This tells us that the sum of all the numbers is <math>64</math>.
 +
 
 +
We want to maximize the value of <math>a\textsubscript{8}</math>, so we set  <math>a\textsubscript{6}</math> and <math>a\textsubscript{7}</math> to <math>8</math> as well.
 +
 
 +
Knowing that we want to minimize numbers and that the range is <math>8</math> , we set <math>a\textsubscript{1}</math>, <math>a\textsubscript{2}</math>, and <math>a\textsubscript{3}</math> equal to <math>a\textsubscript{8} - 8</math>.
 +
 
 +
{<math>a\textsubscript{1}</math>, <math>a\textsubscript{2}</math>, <math>a\textsubscript{3}</math>, <math>a\textsubscript{4}</math>, <math>a\textsubscript{5}</math>, <math>a\textsubscript{6}</math>, <math>a\textsubscript{7}</math>, <math>a\textsubscript{8}</math>} <math>=</math> {<math>a\textsubscript{8} - 8</math>, <math>a\textsubscript{8} - 8</math>, <math>a\textsubscript{8} - 8</math>, <math>8</math>, <math>8</math>, <math>8</math>, <math>8</math>, <math>a\textsubscript{8}</math>}
 +
 
 +
Letting the sum of all the numbers be <math>64</math>, we find that <math>32 + 4a_8 - 24 = 64</math>, which simplifies to <math>4a_8 = 56</math>. Solving, we get <math>\boxed{\text{(D)}\ 14 }</math>. ~By SK80, mod_x for minor edits
 +
 
 +
== Video Solution by OmegaLearn ==
 +
https://youtu.be/xqo0PgH-h8Y?t=848
 +
 
 +
~ pi_is_3.14
 +
 
 +
== Video Solution ==
 +
https://youtu.be/BNa1Ffu517I
  
 
== See Also ==
 
== See Also ==

Revision as of 03:40, 16 January 2023

The following problem is from both the 2002 AMC 12A #15 and 2002 AMC 10A #21, so both problems redirect to this page.

Problem

The mean, median, unique mode, and range of a collection of eight integers are all equal to 8. The largest integer that can be an element of this collection is

$\text{(A) }11 \qquad \text{(B) }12 \qquad \text{(C) }13 \qquad \text{(D) }14 \qquad \text{(E) }15$

Solution 1

As the unique mode is $8$, there are at least two $8$s.

As the range is $8$ and one of the numbers is $8$, the largest one can be at most $16$.

If the largest one is $16$, then the smallest one is $8$, and thus the mean is strictly larger than $8$, which is a contradiction.

If we have 2 8's we can add find the numbers 4, 6, 7, 8, 8, 9, 10, 12. This is a possible solution but has not reached the maximum.

If we have 4 8's we can find the numbers 6, 6, 6, 8, 8, 8, 8, 14.

We can also see that they satisfy the need for the mode, median, and range to be 8. This means that the answer will be $\boxed{\text{(D)}\ 14 }$. ~By QWERTYUIOPASDFGHJKLZXCVBNM

Solution 2

We could express this collection as integers $a\textsubscript{1}$ through $a\textsubscript{8}$, with $a\textsubscript{1}$ being the smallest and $a\textsubscript{8}$ being the largest.

Since the mean is $8$, we know that $a\textsubscript{4}$ and $a\textsubscript{5}$ must also be $8$. If they were not, the other numbers, which are lesser and greater than $a\textsubscript{4}$ and $a\textsubscript{5}$ respectively, would not be able to satisfy the condition that $8$ is the mode.

There are $8$ terms and the mean is $8$. This tells us that the sum of all the numbers is $64$.

We want to maximize the value of $a\textsubscript{8}$, so we set $a\textsubscript{6}$ and $a\textsubscript{7}$ to $8$ as well.

Knowing that we want to minimize numbers and that the range is $8$ , we set $a\textsubscript{1}$, $a\textsubscript{2}$, and $a\textsubscript{3}$ equal to $a\textsubscript{8} - 8$.

{$a\textsubscript{1}$, $a\textsubscript{2}$, $a\textsubscript{3}$, $a\textsubscript{4}$, $a\textsubscript{5}$, $a\textsubscript{6}$, $a\textsubscript{7}$, $a\textsubscript{8}$} $=$ {$a\textsubscript{8} - 8$, $a\textsubscript{8} - 8$, $a\textsubscript{8} - 8$, $8$, $8$, $8$, $8$, $a\textsubscript{8}$}

Letting the sum of all the numbers be $64$, we find that $32 + 4a_8 - 24 = 64$, which simplifies to $4a_8 = 56$. Solving, we get $\boxed{\text{(D)}\ 14 }$. ~By SK80, mod_x for minor edits

Video Solution by OmegaLearn

https://youtu.be/xqo0PgH-h8Y?t=848

~ pi_is_3.14

Video Solution

https://youtu.be/BNa1Ffu517I

See Also

2002 AMC 12A (ProblemsAnswer KeyResources)
Preceded by
Problem 14
Followed by
Problem 16
1 2 3 4 5 6 7 8 9 10 11 12 13 14 15 16 17 18 19 20 21 22 23 24 25
All AMC 12 Problems and Solutions
2002 AMC 10A (ProblemsAnswer KeyResources)
Preceded by
Problem 20
Followed by
Problem 22
1 2 3 4 5 6 7 8 9 10 11 12 13 14 15 16 17 18 19 20 21 22 23 24 25
All AMC 10 Problems and Solutions

The problems on this page are copyrighted by the Mathematical Association of America's American Mathematics Competitions. AMC logo.png